LSAT and Law School Admissions Forum

Get expert LSAT preparation and law school admissions advice from PowerScore Test Preparation.

 Administrator
PowerScore Staff
  • PowerScore Staff
  • Posts: 8916
  • Joined: Feb 02, 2011
|
#24961
Complete Question Explanation

Weaken—CE. The correct answer choice is (A)

This stimulus presents the results of a study which attempted to determine if magnets could help patients with back pain. In the study, one group of patients had magnets applied to their backs, and the other received no treatment. At the end of the study, those who had the magnet treatment reported a significant decrease in their perceived levels of pain, while those who received no treatment reported no reduction. From this, the author concludes that the magnet treatment caused the reduction in pain.

As with many causal relationships, the easiest way to weaken the conclusion is by recognizing an alternate cause. We want to think of a reason, other than the magnates, that the two groups differ. A key factor may be the fact that this was not a blind study. Participants knew if they were receiving the magnet treatment or no treatment by the nature of the study. After all, it would be hard not to notice a magnet on your back. The knowledge that one is receiving treatment is the alternate cause to explain the reported decrease in pain reported by the first group.

Answer choice (A): This is the correct answer choice. As explained above, the patient’s knowledge of the fact he or she is receiving treatment can impact subjective levels of pain. It is for this reason that many studies are “blind” studies where one group receives treatment, while the control group receives a placebo—something that makes them think they could be receiving the treatment. Patients in blind studies do not know if they are being treated or not, so the mere presence of treatment is eliminated as a possible cause for any effects found. Here, since the patients knew they were getting treatment (or not), this answer choice would be an alternate cause which would weaken the argument’s conclusion.

Answer choice (B): This answer choice addresses physician beliefs, and does not address the causal relationship described above. Our goal is to weaken the connection between magnets and patients’ reports of a decrease in pain. A physician’s belief would not help to weaken this connection. Further, the causal argument was comparing magnet use to a complete lack of treatment. The argument does not attempt to compare types of treatment, but rather a type of treatment versus no treatment at all. The study was not attempting to determine what type of treatment is best to manage back pain, but rather could magnets work as a type of treatment to manage back pain.

Answer choice (C): This answer choice does not weaken the conclusion, as the conclusion is specifically focused on “some back pain.” Stating that magnets are not effective on other areas or types of pain is not an effective way to weaken a conclusion which is limited to back pain.

Answer choice (D): This answer choice attacks those who created the study. It sounds plausible that the developers’ beliefs could impact the results of the study. However, as we read through this answer choice, we realize that those researchers with a bias were not actually involved in carrying out the study. Therefore, since the study was not conducted by those with a bias, the biases of the developers would not weaken the conclusion.

Answer choice (E): As explained above, to weaken this causal argument, we likely will need to find some difference between the two groups, other than treatment. This answer choice does not distinguish between the participants in the two groups, but just states that the participants had a wide variety of back pain. It does not tell us, for example, that the first group consisted of patients with low back pain, while the second group consisted of patients with neck pain. It merely states that there were different types. Since it does not allow us to draw a new distinction between the two groups, it is not the correct answer choice.

Get the most out of your LSAT Prep Plus subscription.

Analyze and track your performance with our Testing and Analytics Package.